(Probability/Statistics) Transformation of Bivariate Random Variable

Click For Summary
The discussion focuses on finding the joint probability density function (pdf) of transformed variables Y_1 and Y_2 derived from the original variables X_1 and X_2, given the joint pdf h(x_1, x_2) = 8x_1x_2. The initial attempt to derive the joint pdf by substituting Y_1 and Y_2 directly into the equation resulted in an incorrect answer. The key to solving the problem was identified as the necessity of including the Jacobian determinant in the transformation process. This correction allowed for the proper transformation of the variables, leading to the correct joint pdf. Understanding the role of the Jacobian is crucial in such transformations in probability and statistics.
rayge
Messages
25
Reaction score
0

Homework Statement



Let X_1, X_2 have the joint pdf h(x_1, x_2) = 8x_1x_2, 0<x_1<x_2<1, zero elsewhere. Find the joint pdf of Y_1=X_1/X_2 and Y_2=X_2.

Homework Equations



p_Y(y_1,y_2)=p_X[w_1(y_1,y_2),w_2(y_1,y_2)] where w_i is the inverse of y_1=u_1(x_1,x_2)

The Attempt at a Solution


We can get X_1=Y_1Y_2 and X_2=Y_2. Naively plugging in y, we can get 8y_1y_2^2. However this isn't right according to the back of the book.

I thought it might have to do with finding the marginal distributions of x_1, x_2, but that doesn't seem to lead me anywhere either. Any thoughts welcome!
 
Physics news on Phys.org
rayge said:

Homework Statement



Let X_1, X_2 have the joint pdf h(x_1, x_2) = 8x_1x_2, 0<x_1<x_2<1, zero elsewhere. Find the joint pdf of Y_1=X_1/X_2 and Y_2=X_2.

Homework Equations



p_Y(y_1,y_2)=p_X[w_1(y_1,y_2),w_2(y_1,y_2)] where w_i is the inverse of y_1=u_1(x_1,x_2)

The Attempt at a Solution


We can get X_1=Y_1Y_2 and X_2=Y_2. Naively plugging in y, we can get 8y_1y_2^2. However this isn't right according to the back of the book.

I thought it might have to do with finding the marginal distributions of x_1, x_2, but that doesn't seem to lead me anywhere either. Any thoughts welcome!

You forgot the Jacobian, necessary to transform dx1*dx2 into h(y1,y2)*dy1*dy2
 
Thanks! That was it.
 
Question: A clock's minute hand has length 4 and its hour hand has length 3. What is the distance between the tips at the moment when it is increasing most rapidly?(Putnam Exam Question) Answer: Making assumption that both the hands moves at constant angular velocities, the answer is ## \sqrt{7} .## But don't you think this assumption is somewhat doubtful and wrong?

Similar threads

  • · Replies 2 ·
Replies
2
Views
1K
  • · Replies 15 ·
Replies
15
Views
1K
  • · Replies 5 ·
Replies
5
Views
2K
Replies
4
Views
3K
  • · Replies 11 ·
Replies
11
Views
2K
  • · Replies 1 ·
Replies
1
Views
2K
Replies
5
Views
2K
  • · Replies 5 ·
Replies
5
Views
8K
  • · Replies 2 ·
Replies
2
Views
2K
  • · Replies 4 ·
Replies
4
Views
2K